SlideShare una empresa de Scribd logo
1 de 26
Descargar para leer sin conexión
Academy of Fashion & Design
                             Email: contact@afdindia.com
                                  Ph: 08800337062




            Get into premier design school In India & Abroad with AFD
                              http://www.afdindia.com
                                          



                                                            

                               MASTERS IN DESIGN
                              GENERAL ABILITY TEST

Time allowed:- 2 hrs                                                               Total
        Question:- 150
This test comprises the following sub-tests.


Quantitative ability
English comprehension
English Communication
Analytical ability
General Awareness


Each question carries one mark.
Answers are required to be marked only on the OMR/ICR Answer-Sheet, which shall be
       provided separately.
For each question, four alternative answers have been provided out of which only one is
       correct. Darken the appropriate circle in the Answer-Sheet by using Ball Pen only
       on the best alternative amongst a), b), c) or d).
                                Section-1-Quantitative ability

Q1.    Three bells commence tolling together and they toll after 0.25, 0.1 and 0.125 sec.
       After what interval will they again toll together?
       a) 1.5 sec             b) 2.5 sec               c) 0.5 sec          d) 4.2 sec

Q2.    Find the smallest fraction which, when added to EMBED Equation.3          gives a
whole number.
       a) 37 / 40             b) 35 / 39              c) 32 / 40             d) 45 / 35


Q3.    If 5 = 2.24, then what is the value of EMBED Equation.3 ?
       a) 1.48               b) 1.58              c) 1.68                    d) 1.89


Q4.    Find the value of a 3 + b 3 + c 3 – 3abc when a + b + c = 9 and a 2 + b 2 + c 2 =
       29.
       a) 9                   b) 3                 c) 27                 d) 81
       e) None of these

Q5.    If we write the numbers from 1 to 201, what is the sum of all the odd numbers?
       a) 10,201              b) 10,220             c) 9,102                d) None of
       these
Q6.    The sum of two numbers is 20, and their difference is 2 EMBED Equation.3 , find
       the ratio of the numbers.
       a) 8 : 10              b) 2 : 8              c) 9 : 7                d) 10 : 12

Q7.    In an exam, the average was found to be 50 marks. After deducting computational
       errors the marks of


           the 100 candidates had to be changed from 90 to 60 each and the average came
       down to 45 marks. The total number of candidates who took the exam were:
       a) 300                 b) 600               c) 200                d) 150
       e) None of these

Q8.    A man buys milk at 60 P per liter adds one – third of water to it and sells the
       mixture at 72 P per liter. The profit per cent is _____________.
       a) 45%                  b) 60%                   c) 72%          d) 75%

Q9. Two equal amounts of money are deposited in two banks, each at 15 % per annum,
for EMBED Equation.3 years and
       5 years respectively. If the difference between their interests is Rs. 144 each sum is
Rs __________.
      a) 460/-                b) 500/-               c) 640/-                  d) 720/-

Q10. I give a certain person Rs. 8000 at simple interest for 3 years at EMBED Equation.
     3 p.c. How much more should I have gained had I given it at compound interest?
     a) 148.45/-             b) 138.38/-            c) 134.89/-           d) 134.25/-
Q11. A sum of Rs. 6.40 is made up of 80 coins, which are either 10–paise or 5–paisa
     coins. How many are of 5 P.?
     a) 58                b) 64                c) 32                d) 25

Q12. A and B can finish a field work in 30 days, B and C in 40 days while C and A in
     60 days. How long will they take to finish it together?
     a) 26 EMBED Equation.3 days                    b) 39 days          c) 25 EMBED
     Equation.3 days                d) 25 days
Q13. If 3 men with 4 boys can earn Rs. 756 in 7 days, and 11 men with 13 boys an earn
     Rs. 3008 in 8 days, in what time will 7 men with 9 boys earn Rs. 2480?
     a) 8 days              b) 5 days               c) 12 days          d) 10 days

Q14. Pipe A can fill a tank in 20 hours while pipe B alone can fill it in 30 hours and pipe
     C can empty the full tank in 40 hours. If all the pipes are opened together, how
     much time will be needed to make the tank full?
     a) 7 EMBED Equation.3 hrs.                    b) 1 EMBED Equation.3 hrs.
            c) 17 EMBED Equation.3 hrs.                    d) 77 EMBED Equation.3
     hrs.

Q15. Compare the rates of two trains, one travelling at 45 km an hour and the other at 10
     m a second.
     a) 5 : 4              b) 6 : 4                c) 2 : 6              d) 4 : 8

Q16. Find the length of a bridge which a train 130m long, travelling at 45 km an hour,
can cross in 30 sec.
       a) 396 m            b) 249 m              c) 245 m                d) 234 m

Q17. A man can row 6 km/hr in still water. When the river is running at 1.2 km/hr, it
     takes him 1 hour to row to a place and back. How far is the place?



       a) 2.66 km             b) 2.85 km             c) 88.2 km            d) 2.88 km

Q18. If 2 liters of water are evaporated on boiling from 8 liters of sugar solution
     containing 5 % sugar, find the percentage of sugar in the remaining solution.
     a) 6 EMBED Equation.3 %               b) 6 EMBED Equation.3 %                c) 5
     EMBED Equation.3 %                    d) 2 EMBED Equation.3 %

Q19. The age of a man is 4 times that of his son. 5 yr. ago, the man was nine times as old
     as his son was at the time. What is the present age of the man?
     a) 64 yrs               b) 49 yrs              c) 32 yrs             d) 45 yrs
What is the greatest number that will divided 38,45 and 52 and leave as remainders 2,3 and
4 respectively?
5                             b) 6                          c) 7
d) 8
Find the greatest number, which will divide 410, 751 and 1030 so as to leave remainder 7
       in each case?
30                    b) 28                         c) 31                       d) 29
The numbers 11284 and 7655, when divided by a certain number of three digits, leave the
      same remainder. Find that number of three digits.
191                b) 130                          c) 131                      d      )
190
Find the greatest number of six digits which, on being divided by 6,7,8,9 and 10 leaves
       4,5,6,7 and 8 as remainders respectively.
997918                        b) 997920                   c) 887920
d) 887918
Find the greatest number less than 900, which is divisible by 8,12 and 28.d
540                   b) 640                          c) 740                      d      )
840
What greatest number can be subtracted form 10,000 so that the remainder may be divisible
       by 32, 36, 48 and 54? B
9186                  b) 9136                      c) 3156                       d      )
8146
Find out whether (3x – 1) is a factor of 27x3 – 9x2 – 6x + 2 by the above rule.
(3x-2)                b) (3x-1)                       c) (2x-1)                   d      )
(3x-2)

What should be subtracted from 27x3 – 9x2 – 6x – 5 to make it exactly divisible by (3x –
      1)?
-5                           b) -6                      c) -7
d) -8

If x + _1_ = 2 , the value of x2 + _1_ = ?
               X                             X2
6                              b) 5                          c) 1
d) 2

If X + Y = 3, XY = 2; find the value of X3 – Y3 ?
6                             b) 7                           c) 3
d) 4
If x = 12, y = 4; find the value of (x+y)x/y
4096                     b) 3096                       c) 2096                         d       )
1096


                          Academy of Fashion & Design
                              Email: contact@afdindia.com
                                   Ph: 08800337062




            Get into premier design school In India & Abroad with AFD
                              http://www.afdindia.com
                                          



                                                                 

                            Section-2- English Comprehensive

Directions (Q31-35):- Read the passage and answer the questions that follow:

       Mrs. Loisel now learned what it was like to be really poor. She made up her mind
       to face it and played her part bravely. This terrible debt had to be paid and she
       would pay it. The maid was dismissed; the flat was given up and they moved into a
       garret. She did all the rough household work; washed up after meals and ruined her
       finger nails scrubbing dirty dishes and pans. She did all the washing and hung it out
       on the line to dry. Every morning, she carried the rubbish down to the street and
       brought the water, pausing for breath at the top of each flight of stairs. Dressed like
       a working woman, she went with her basket on her arm to the greengrocer, the
       grocer and the butcher, bargaining, arguing and fighting for every penny.




Her husband spent his evenings, working at some shopkeepers’ account, and at night, he
would often copy papers at a few pennies a page.

Thus life went on for ten years.

At the end of that time, they had paid off every thing to the last penny, including the interest
on the loan.
Mrs. Loisel now looked like an old woman. She had become a typical poor man’s wife,
rough and coarse. Her hair was neglected, her dress was untidy, and her hands were red.
But now and then, when her husband was at the office, she would sit by the window and
her thoughts would go back to that far away evening, the evening of her beauty and her
success. What would have been the ends of it if she had not lost the necklace? Who could
say? How strange and varied are the chances of life. How small a thing can save or ruin
you.

One Sunday, she went for a walk in the Champa Elysees and she caught sight of a lady
with a child. She recognized Mrs. Forestier, who looked as young, as pretty and as
attractive as ever. Mrs. Loisel felt a wave of sadness pass over her. Should she speak to
her? Why not? Now that the debt was paid, why should she not tell her the whole story?
She went up to her

       “Good morning, Jeanne.”

Her friend did not recognize her and said, “I’ m afraid I don’t know you; you must have
made a mistake.”

               “No, I am Mathilde Loisel.”
               Her friend uttered a cry.
               ‘Oh’ my poor dear Mathilde! How you have changed!”
               “Yes, I have been through a very hard time since I saw you last.”
               “What do you mean?”
               “You remember the diamond necklace you lent me to wear?”
               “Yes. Well?”
               “Well, I lost it.”
               “I don’t understand. You brought it back to me.”

“What I brought you back was another one, exactly like it and for the ten years, we have
been busy paying for it. You must understand that it was not an easy matter for people like
us, who hadn’t a penny? However, it’s all over now. I can’t tell you what a relief it is!”

               Mrs. Forestier stopped dead.

               “You mean that you bought a diamond necklace to replace mine?”

“Yes. And you never noticed it? They were certainly very much alike,”
She smiled with pride and satisfaction. But Mrs. Forestier seized both her hands in great
distress.
“Oh, my poor, dear Mathilde1 Why? Mine was only imitation. At the most it was worth
five hundred francs!”
Q31. Mrs. Loisel was not a well off lady but still, she took to pay back the huge amount
of necklace
        a) In a half – hearted way                         b) In a reluctant manner
        c) Quite unwillingly                               d) In a determined way
        e) Under compulsion of law

Q32. Entire money was paid up by her without excuse but
     a) It took ten years for Loisel to clear off the dues in installments
     b) It took ten years for them to save that much amount to pay back
     c) It took lesser period in paying back the money.
     d) They had not paid back in full the amount of necklace even during a period of ten
     years
     e) They were running short of a little amount to pay back the amount in full at the
     end of ten years.

Q33. Mrs. Loisel struggled very hard to pay back the amount and it.
     a) Had made her weak and ill                        b) Had no effect on her health
     c) Had made her demoralised and ill humoured        d) Had bereft her of her
     memory
     e) Had affected her looks

Q34. When Mrs. Loisel met Mrs. Forestier she
     a) Told her that she had lost the necklace but promised to return another exactly like
     it
     b) Admitted that she had replaced Mrs. Forestier’s diamond necklace with a
     necklace of imitation diamonds.
     c) Told her that the necklace she had returned was not the same that she had
     borrowed
     d) Accused Mrs. Forestier of being a cause of ten years sufferings
     e) Begged her pardon to avoid talk about the necklace that has caused her so much
     worry

Q35. Mrs. Forestier told Mrs. Loisel to the effect that.
     a) The necklace that was returned was made of imitation diamonds
     b) The necklace borrowed by Mrs. Loisel was not of real diamonds
     c) The necklace returned valued just 500 francs
     d) The necklace that she had returned valued less than 500 francs
     e) The necklace she had lent was of real diamonds

Direction (Q36 - 40 ): Read the following passage carefully and answer the questions
given below it.
Imagine a ship built to sail amongst the ice at the North Pole. The ship has a hull two feet
thick. This hull is shaped so that when the ice closes in on the ship she is pushed up – like
a piece of soap in your hand and drifts in the top layer of the ice. A ship like this was built
for the great Norwegian explorer Fridjof Nansen.

One day, while he was walking by the sea, Nansen found a piece of Russian wood.
Obviously the wood had drifted south across the Arctic Circle from Siberia. Nansen
thought that if a piece of wood could make the journey so could he. If he had a ship which
rested on the ice he would he able to drift across the Arctic Sea. And so that ‘Fram’ (which
means forward) was built.

In June 1893, Nansen left Oslo with a crew of 13 men and 30 dogs for the sledges. Many
people thought his plans were mad and said that he was going to meet his death. By Sp.
The ‘Fram’ had reached Cape Chelyuskin, the most northerly point of Siberia. Here, the ice
became 30 ft thick and began to close in on the ‘Fram’. But just a Nansen had said the ship
rose up, came to rest on the ice, and began to drift. For a year she




drifted northwards, then slowly westwards. Nansen then decided to rush to the North Pole
with his companion Hjelmar Johansen. At the time, no explorer had ever reached the Pole.

They set off with 28 dogs, 3 sledges, 2 small boats, a tent, sleeping bags and food. By
April 1895 they were within 200 miles of the Pole.

This was nearer to the Pole than any other explorer had ever been. But conditions were
terrible. The temperature dropped to 40 degrees below freezing point. The dogs became
exhausted. Their cloths froze on their bodies. In the end they had to turn back. The two men
and their dogs travelled for four months through ice and snow. Frozen, exhausted and with
sinking spirits, they struggled on through mile after mile of snow.

Once Nansen was nearly killed by animals. On another occasion, they tied their small boats
to an iceberg and climbed up to make observations. The boats broke loose and began to
drift. Nansen knew that they would not be able to go on without the boats. Still wearing
this heavy cloths, he dived into the freezing water and swam after the boats. Then, after he
had got them back, Nansen boat was nearly destroyed by an animal.

At last, they reached Franz Josef Land, which is still inside the Arctic Circle. They decided
to spend the winter there because it would have been dangerous to travel in a Pole winter.
They built a hut of stone and covered it with animals’ skins and then, settled down for a
long cold wait. They spent the winter hunting, talking and writing. In this way, they
stopped themselves from going mad with loneliness.
In May, when spring came, they set out again on their march Southwards. A month later,
they walked into a party of British explorers. The British had been in the Arctic for two
years and were waiting for their ship to take them home. The ship arrived and took Nansen
and Johansen back to Norway.

But what had happened to the “Fram”? There was no word. Her movements were as much
a mystery as those of Nansen and Johansen had been. Astonishingly, a week after
Nansen’s return, the ‘Fram’ drifted back to Norway – exactly three years after she had let.
Nansen was given a hero’s welcome and so were the men of the ‘Fram’.

Q36. For going on journey across the Arctic, Nansen got an idea of a particular ship,
especially designed
       for the purpose because
        a) He must travel by a newly deigned ship to go on exploration mission.
        b) He must have a ship that would sail around the frozen Arctic.
        c) He must have a ship that would plough through the ice
        d) He must have a ship that would slide the top layer of the ice
        e) He must have a ship that would rest on the ice and drift.

Q37. How did Nansen plan to go on exploration mission across the Arctic
     a) He planned to go to Russia across the Arctic Sea
     b) He planned to sail towards the North – Pole
     c) He planned to sail north – west toward the Pole
     d) He planned to proceed Northward on the ice
     e) He planned to go across the Arctic from north to south

Q38. What happened when Nansen and Johansen tried to reach the North Pole
proceeding westward?

       a) They lost the way and were misled                 b) They reached the Pole but
       could and stay there
       c) They reached the Pole and passed the winter there
       d) They approached closer to the Pole but were forced to keep away by weather
       extremities

Q39. What happened to Nansen and Johansen while they were on return journey?
     a) They met unknown explorers who played host to them
     b) They met British explorers who offered them lift to Norway
     c) They met the British explorers who took them home in their ship
     d) They met the British explorers who took them home in their ship – the “Fram”
e) They got a ship that had just landed a party of explorers and reached home.

Q40. What happened to ‘Fram’ after it was abandoned by Nansen and Johansen?
     a) The ship was destroyed by the snowstorm         b) Returned to Norway after a
     very long time
     c) Returned to Norway shortly after Nansen         d) Nothing was heard of it
     e) The ship was remodeled by some other explorers and taken away by them.

Directions:       Below is given a passage followed by several possible followed by several
              possible inferences which can be drawn from the facts stated in the passage.
              You have to examine each inference separately in the context of passage and
              decide upon its degree of truth or falsity and mark: (a) if it is definitely true. (b)
              if it is probably true. (c) if the data are inadequate. (d) if it is probably false. (e)
              if it is definitely false.

        The serious accident in which a person was run over by a car yesterday has again
        focussed attention on the most unsatisfactory state of street lighting. No one expects
        side roads to be provided with the same standard of lighting, as a main road, but
        unless the council is prepared to make good its promise as regards road lighting, it
        will only be a question of time before there are further and perhaps fatal accidents.

It seems that council has promised to improve the state of lighting on side roads.
There will not be a single accident on roads if roads are satisfactorily illuminated.
The accident that occurred was fatal.
Several accidents have so far taken place because of unsatisfactory lighting.
The accident occurred at night.


Direction: Read the following passage and answer the question:

There are several reasons for a headache. Physical, emotional and mental factors,

anxiety and tension are a few. Sometimes, headache can be a signal of an underlying

disease. More than medicines, yoga therapy eminently suits any need. Yoga is a

comprehensive mode of culturing the body and the mind. Using an ‘Intergrated

Approach of Yoga’, the yoga Research Centres have been able to cure some tough

headaches. The integrated approach includes breathing, asanas, pranayama,

meditation and devotional sessions.
Yoga asanas, especially the ones imitating the natural postures of animals, have a
tremendous tranquilising effect, without having t depend on common drugs. Pranayama
inhibits random agitation in Pranic (energy) flows in Pranamayakosa, stabilising the
autonomic nervous system. Dhyana and Samadhi culture the mind to relax it. This approach
alters the reaction of an individual to headache. By interrupting the vicious cycle of pain –
agony – pain, it prevents headache from becoming a crippling problem.
Through asanas that calm you , the pranayama exercises that inhibit random energy flows
and the meditation that cultivates and relaxes your mind, yoga offers a holistic from of pain
relief. It stops you from becoming locked in the vicious circle of pain-anxiety- pain.
Chronic pain: Chronic pain essentially is imbalance in prana (energy). This imbalance
initially manifests only as functional abnormality like insomnia, lack of enthusiasm, fatigue,
increased irritability and lack of concentration. Over the years, the imbalance settles in an
organ. Chronic pain may sometimes be just a long standing muscle spasm, which later on
amy give rise to organic changes in the form of chronic inflammation.


The main theme of the passage is
                Headache
                Yoga Asanas
                Chronic Pain
                Relief from Headache

Reasons for headache are
                  Physical emotional and mental.
                  Lack of concentration
                  Too much of TV
                  Carry any disease.

What suits more than medicines?
                   Asanas
                   Homeopathy
                   Integrated approach
                   Yoga therapy

What cultures the mind?
                   Drugs
                   Medicines
                   Pain and agony
                   Dhyana and samadhi

What prevents headache from becoming a crippling problem?
                  By attiring the approach
                  By stabilizing the nervous system
                  By interrupting the flow of energy.
                  By interrupting the viscious cycle of pain – agony – pain
Asanas calm you and Pranayama inhibit
             Viscous cycle of pain
             Stabilizes the mind
             Relaxes the mind
             Inhibits random energy flow.

Yoga relaxes the mind and offer
               Holistic form of pain relief.
               Alters the reaction
               Dependence on common drugs
               To culture body and mind

The disturbance in the energy level may lead to ____
               Chronic pains
               Imbalance situation
               Pain and agony
               Distraction

Chronic pain can sometimes be
               Fatigue
               Lack of concentration
               A long standing muscle spasm
               Organic changes / chronic inflammation

For more than 3 decades, I achieved great success as a lawyer, till a stroke left my right side
totally paralysed. Despite the doctor’s encouragement, I was consumed by rage and self-
pity. I yearned to be active again. But what could a middle-age cripple like me do ? One
day, glancing at some paintings I owned, I thought suddenly, “what about painting” ? In
fact, I had always wanted to paint, but had never had the time. Now, I had plenty of time. In
the last 25 years, I have completed 300 paintings—one of them appeared on the cover of
the Reader’s Digest. The stroke, I realize, has helped me develop a latent talent and enjoy
life.

55. Which one of the following is the correct statement ?
While in the hospital, the author was—
(A) angry with himself for falling ill.
(B) relieved at the successful treatment.
(C) frustrated at his helpless situation.
(D) resentful at being hospitalized.

56. Why did the author consider himself a cripple?
(A) He could not go back to work.
(B) He could no longer use his right hand.
(C) He could not use his time properly.
(D) He could not lead an active life.

57. Which one of the following is the correct statement ?
The paralytic stroke helped the author—
(A) to face challenges in life successfully.
(B) to realize his latent talent.
(C) to learn a new hobby.
(D) to earn more money.

PASSAGE
“Westward Ho !” we shouted as the sail of our crudely constructed raft, the Kon-Tiki
caught the wind. The sail quickly filled and the Kon-Tiki began to move. The six of us
were off to our great adventure.
As night fell, the troughs of the sea grew gradually deeper and our first duel with the
elements began. Each time we heard the sudden deafening hiss of a roller closeby and saw
a white crest come towards us out of the darkness, we held on tight and waited for the
worst. But invariably the Kon-Tiki calmly swung up her stern and rose skyward
unperturbed.

58. What does the word ‘duel’ in the passage mean ?
(A) A battle
(B) A fortification
(C) A two-side contest
(D) Divided in two

59. Which one of the following is correct ?
When big waves struck the raft the six people in it.
(A) Started crying.
(B) Showed courage and patience.
(C) Acted in a rash manner.
(D) Showed passiveness.

60. How was the Kon-Tiki’s performance on the high seas ?
(A) Very shaky
(B) Extremely poor
(C) Stable and resolute
(D) Unpredictable
Academy of Fashion & Design
                            Email: contact@afdindia.com
                                 Ph: 08800337062




            Get into premier design school In India & Abroad with AFD
                              http://www.afdindia.com
                                          



                                                             

                          Section-3 English Communication

Directions (Q.61-65): Choose the alternative, which is same in meaning to the key word:-

AURICULAR
    a) Nose                  b) Eyes                c) Ears               d) Heart

AVOWAL
    a) Sacred oath           b) Open declaration    c) Harsh Sound        d) Stern denial

COGNATE
    a) Known                 b) Of common origin c) Wheel shaped          d) Substantial

FRET
       a) Worry              b) Behave              c) Pine-heartedd) Regret

JOWL
       a) Jaw                b) Fiesta              c) Bellowing          d) Small Jar
Directions (Q.66 - 70): Choose the correct one, which is opposite in meaning to the key
word:-

WARILY
    a) Negligently           b) Timidly             c) Vigilantly         d) Noisily

INTIMATE
     a) Proclaim             b) Personal            c) Apparel            d) Preclude

MODISH
    a) Vulgar                b) Unfashionable       c) Shabby             d) Immodest
WAG
       a) Critic               b) Skunk                c) Wit                  d) Dullard

PROLIX
     a) Clever                 b) Punctual             c) Charitable           d) Precise


Directions (Q.71 - 75): Choose the appropriate preposition:


Mike has been ill ________ a long time. He has been in hospital ________ Oct.
      a) on                  b) to                 c) since                            d) for

The office is very small. There’s space only ______ a desk and chair.
       a) to                   b) since                     c) for                     d) on

Sue went ________ Mexico last year.
      a) for               b) since                             c) to                  d) on

Don’t open the door of the train __________ the train stops..
       a) for                  b) Until                      c) Since
d) on

Where have you been? I’ve been waiting for you _________ twenty minutes.
      a) for                b) Until                    c) Since
d) to

Directions (Q76-80):- Some parts of each of the following sentences are underlined. Either
one underlines part is not acceptable in standard English or there is not error. With regard to
error you are concerned with the underlined part only.

     EMBED Equation.3    Ram, Sham, Hari EMBED Equation.3                          differences
EMBED Equation.3 EMBED Equation.3 settled. EMBED Equation.3 .

       EMBED Equation.3 ask EMBED Equation.3 principal if I EMBED Equation.3
have one day EMBED Equation.3 . EMBED Equation.3 .

       EMBED Equation.3 machine worked EMBED Equation.3 EMBED Equation.3
many days and we EMBED Equation.3 rest at all. EMBED Equation.3 .

Trios EMBED Equation.3 EMBED Equation.3 boys EMBED Equation.3 singing
EMBED Equation.3 well that everyone was happy. EMBED Equation.3 .
EMBED Equation.3 principal EMBED Equation.3 with his colleagues EMBED
Equation.3 in EMBED Equation.3 car. EMBED Equation.3 .

       FILL IN THE BLANKS:- (81-90)

My father is not a t home _________
            a) presently            b) first now                  c) Nil

I have read ___________ book of this library
            a) each        b) every                      c) Nil

He is stronger than anybody _________
            a) else          b) here           c) Nil

__________ European likes to trand much
          a) an             b) a               c) Nil

My young son goes to _______ school
          a) an            b) a                c) Nil

I have got ___________ good health
            a) an           b) the             c) Nil

I passed my holidays _________ the village of my uncle
           a) at            b) in           c) Nil

He will come back ________ a week
           a) Nil          b) within           c) in

Divide this apple ___________ the two boys.
            a) between      b) among       c) Nil

The hunter shot the bird _________ a gun,
            a) with           b) by            c) from
Academy of Fashion & Design
                            Email: contact@afdindia.com
                                 Ph: 08800337062




            Get into premier design school In India & Abroad with AFD
                              http://www.afdindia.com
                                          



                                                             

                              Section 4- Analytical Ability

Directions (Q. 91 –95): Read the information and answer the questions given below the
information

There is a group of six persons – A, B, C, D, E & F in a family. They are psychologist
Manager, Lawyer, Jeweler, Doctor and Engineer.
The doctor is the grandfather of F. Who is a Psychologist. The Manager D is married to A.
C is the Jeweler is married to a Lawyer. B is the mother of F and E. There are two married
couples in the family.

What is the profession of A?
        a) Doctor       b) Engineer         c) Psychologist        d) Manager     e      )
None of these

How is A related to E?
       a) Father       b) Grand Father       c) Mother             d) Sister      e      )
None of these

How many male members are there in the family?
     a) 3                                 b) 4                      c) 2
     d) information insufficient          e) None of these

What is the profession of E?
       a) Doctor         b) Engineer         c) Psychologist       d) Manager     e      )
None of these

Which of the following is one of the married couple?
a) A-B            b) A-C                 c) A-D                  d) F-E          e      )
None of these

Direction (Q96-100): These question consist of a question and two statements numbered I
and II given below it. You have to decide whether the data provided in the statements are
sufficient to answer the question.

Read both the statements and give your answer as follows:
       If the data in statement I alone is sufficient to answer the question
       If the data in statement II alone is sufficient to answer the question
       If the data in either statement I or II is sufficient to answer the question.
       If the data in statement I and II together are not sufficient to answer the question
       If the data in statement I and II together are necessary to answer the question

Who scored the highest among A,B,C,D and E?
B scored more than D, but not as much as C.
E scored more than C but not more than A.
       a) B             b) A                c) D                       d) C            e) E

How many boys are there in the class?
                Mita’s rank among the girls is 5th from the top and her rank in the class is
                9th from the bottom.
            Number of boys in the class is twice the number of girls.
       a) C         b) A                  c) B                   d) D                  e) E

Who is the immediate right of P among five persons P, Q, R, S and T facing North?
              R is third ot the left of Q and P is second to the right of R.
              Q is the immediate left of T who is second to the right of P.
       a) C             b) A                           c) B                  d) D
e) E

Z is in which direction with respect to X?
               Y is to the South of x and Z is to the East of P which is to the North of Y
               P is to the South of x
        a) D              b) C                c) B                    d) A            e) E

How is P related to N?
               N is sister of M, who is son of Q whose wife is P.
               M is brother of N and son of Q whose wife is P.
       a) C              b) D                       c) B                        d) A
e) E
Pointing to a photograph, a person tells his friend “She is the grand daughter of the elder
brother of my father ”. How is the girl related to this man in the photograph
        a) Niece         b) Daughter            c) Daughter-in- law       d) Mother
e) None of these

Direction (Q102 - 105): These questions are based on the following diagram in which the
triangle represents the uneducated person, Circle represents young person, and rectangle
represent employed person.




Which number represents young uneducated & employed person ?
      a) 4                 b) 3             c) 5             d) 2
e) None of these


Which number represents educated, employed and young person.
      a) 2                  b) 3            c) 1                        d) 4            e       )
None of these

Which number represents young, educated and unemployed person?
      a) 3                 b) 1              c) 7              d) 6                     e       )
None of these

Which number represents young uneducated & unemployed person? (Check young should
be there or not)
        a) 3               b) 5            c) 6              d) 4         e     )
None of these

Direction (Q106 - 110):- Answer these questions referring to the symbol-letter number
sequence given below:
        NOPQYBZARSHIJKLMTUVGFEWXDC
If letters of the above given series are written in reverse order then which letter will be third
to the left of eighteenth letter from your right?
         a) Z                      b) F             c) I                d) L             e      )
None of these
What will come in place of question mark (?) in the following series?
      NDP, QWB ,ZFR ?
      a) SVJ                 b) AFS            c) IVS                d) SFA              e       )
None of these

Which of the following is fifth to the right of thirteen letter from your left?
      a) U                     b) J                 c) S                 d) Z            e       )
None of these

If every alternate letter starting from O is replaced with odd numbers starting from 1, which
letter or number will be third to the left of tenth letter from your right?
         a) 15                    b) L               c) K                 d) 13       e     )
None of these

If it is possible to make a meaningful word from the eighth, sixteenth, seventeenth and
twenty-second letters from your left in the given series, which will be the first letter of that
word? If no such word an be formed your answer would be X, and if more than one such
word can be formed answer is P?
          a) M                  b) T              c) X                 d) P              e       )
None of these


In row of 16 boys when Karan was shifted by 2 places towards the left he became 7th from
the left end. What was his earlier position from the right ?
         a) 8th                 b) 9th                   c) 7th           d) 10th
e) None of these

Direction (Q 112-113):- In each of the following find the figure from the Answer set (i.e.
1, 2, 3, 4 and 5 ) which will continue the series given in the problem set (i.e. A, B, C, D,
and E)

PROBLEM FIGURES                                          ANSWER FIGURES




          A        B       C       D      E                         1       2       3        4
5

        a) 2                    b) 3               c) 1                  d) 5            e       )
        None of these
PROBLEM FIGURES                                        ANSWER FIGURES
       A     B    C               D      E          1       2    3      4    5

      a) 5                    b) 3               c) 4                d) 1            e      )
None of these

If south-east becomes north, north-east becomes west and so on, what will west become?
        a) north-east                    b) north-west                     c) south-east
        d) south-west                    e) None of these

A bus for Delhi leaves every forty minutes from a bus stand. An inquiry clerk told a
passenger that the bus had already left ten minutes ago and the next bus would leave at 9.45
a.m. At what time did the inquiry clerk give this information to the passenger?
       a) 9.05 am                       b) 9.15 am                              c) 9.25 am
       d) 9.35                          e) None of these


Directions (Q116 - 120): Each of the following questions consists of five figures marked
        A,B, C, D and E
called the Problem Figures followed by five other figures marked 1, 2, 3, 4, and 5 called the
        Answer Figures.
Select a figure from amongst the Answer Figures which will continue the same series as
        established by the
five Problem Figures.
Academy of Fashion & Design
                              Email: contact@afdindia.com
                                   Ph: 08800337062




              Get into premier design school In India & Abroad with AFD
                                http://www.afdindia.com
                                            



                                                                


                              Section 5- General Awareness


Which India State advertise itself as “ Never out of season”?

kerela
Himachal
Uttaranchal
Goa

Which airline has a heart painted at the entry of each of its aircraft cot the company HQ is at
lone field in Dallas?

Quantas
Gulf Airways
Southwest Airlines
Air china.


Which memorable character made his appearance on the comic strip on October 4th 1950.

Mickey Mouse
Snoopy
Fan tom
Batman
The ‘Jeans that Built America’?

Diesel
Levi’s
Mudd
Lee

Find the odd one out

Kathakali
Mohini Attam
Bharat Natyam
Chakri

The first cotton mill of India was set up at …….

Surat
Mumbai
Kanpur
Ahmedabad

Which of the following is the capital of “the land of midnight sun”?

Vienna
Oslo
Helsinki
Pragne


   Where would you find the Indian Textile Institute?

Surat
KAnpur
Pune
Mysore

The study of the heavenly bodies is
       a) Astronaut’s        b) Astronomy            c) Astrology      d) Aewnautics


 130. World Literacy Day is on?

         a) Feb. 8th          b) Jan. 18th           c) Sept. 8th      d) Feb. 18th
131. World Trade Organisation (WTO) came into existence in:
      a) 1996               b) 1998             c) 1995                    d) 1997

 132. The World Cup Cricket is held once in every
         a) 4 years           b) 5 years            c) 2 years             d) 3 years
 133. Baseball is the national game of
         a) England           b) Kenya              c) USA                 d) South
Africa


 134. The Flag of which colour is used by an enemy while surrendering?
       a) White              b) Black              c) Red                  d) Yellow


 135. Which of the following ranks the highest in the Indian Army?
       a) Colonel     b) Lieutenant General          c) Major              d) Captain


 136. According to Hindu Mythology, who is the creator of the word?
       a) Brahma     b) Vishwakamla               c) Vishnu                d) Mahesha


 137. Which one of the following is not an ancient scripture?
       a) Rigveda            b) Mundaka              c) Upnishada          d)
Arthashastra


 138. Camel can store extra water in the_______.
       a) Rumen              b) Coelom              c) Stomach             d) None of
these
 139. The smallest State in area is:
       a) Kerala               b) Tripura           c) Punjab              d) Goa

 140. ‘Eat Healthy Think Better’? Tagline of which company.


Parle
Brittania
Priya Gold
Cherry merry


 141. Tradus.in has launched an online book store at TradusBooks. in, offering book
      enthusiasts a wide assortment of titles, ease of shopping and free shipping of books
across India. Tradus.in is a sister concern company of …

       a. Ibibo Web Pvt Ltd
       b. Yahoo
       c. Times Shopping
       d. 160by2.com

  142. Harry Potter ends up marrying

       a.Hermoine
       b.Ron
       c.Gennie
       d.Snape

   143. What is the amount that Future Group’s unit Pantaloon Retail planning to raise
from
        its IPO?

       a. ` 600 cr
       b. ` 650 cr
       c. ` 700 cr
       d. ` 750 cr

  144. HDFC Trustee Co Ltd acquired 2.44 per cent stake in which textiles and apparels
       maker company in August?

       a. Raymond
       b. Grasim
       c. Peter England
       d. Armani

   145. How many circles have been allotted to Vodafone for 3G Mobile services?

       a. 9
       b. 11
       c. 14
       d. 17

   146. Spice Jet has been acquired by…

       a. Reliance ADA Group
       b. Mahindra n MahindraGroup
       c. Sun Group
d. Larsen & Tourbo

   147. Recently, which one of the following was included in the UNESCO's World
Heritage list?

       a. Dilwara Temple
       b. Kalka-Simla Railway
       c. Bhiterkania Mangrove Area
       d. Vishakapatnam to Araku valley railway line

  148. Which of the following is also known as “ Silicon valley of India”?
      a. Hyderabad
      b. Bangalore
      c. Chennai
      d. Noida

  149. Peace Palace is the official headquarters of..........
       a. WHO
       b. FAO
       c. ICJ
       d. IBRD

  150. Find the odd one out.
       a. DOS
       b. WINDOWS
       c. LINUX
       d. LISP

Más contenido relacionado

La actualidad más candente

Ancient Egyptian Designs
Ancient Egyptian DesignsAncient Egyptian Designs
Ancient Egyptian DesignsFrank Curkovic
 
Introduction to Visual Culture
Introduction to Visual CultureIntroduction to Visual Culture
Introduction to Visual CultureDeborahJ
 
Irregular forms
Irregular  formsIrregular  forms
Irregular formsBarkha
 
Chapter 9 pattern and texture
Chapter 9 pattern and textureChapter 9 pattern and texture
Chapter 9 pattern and textureTracie King
 
Introduction to Visual Culture
Introduction to Visual CultureIntroduction to Visual Culture
Introduction to Visual CultureDeborahJ
 
Introduction to islamic architecture
Introduction to islamic architecture Introduction to islamic architecture
Introduction to islamic architecture RUSHALI SRIVASTAVA
 
Geometry of Islamic Architecture
Geometry of Islamic ArchitectureGeometry of Islamic Architecture
Geometry of Islamic ArchitectureGuneet Khurana
 

La actualidad más candente (20)

Nift 2009 cat sample question papers
Nift 2009 cat sample question papersNift 2009 cat sample question papers
Nift 2009 cat sample question papers
 
NIFT study material creative ability test 2
NIFT study material creative ability test 2NIFT study material creative ability test 2
NIFT study material creative ability test 2
 
Nift ug design CAT sample paper2
Nift ug design CAT sample paper2Nift ug design CAT sample paper2
Nift ug design CAT sample paper2
 
Ancient Egyptian Designs
Ancient Egyptian DesignsAncient Egyptian Designs
Ancient Egyptian Designs
 
nid pg sample questions of the admission test
nid pg sample questions of the admission test nid pg sample questions of the admission test
nid pg sample questions of the admission test
 
Visit to Craft Museum
Visit to Craft MuseumVisit to Craft Museum
Visit to Craft Museum
 
Introduction to Visual Culture
Introduction to Visual CultureIntroduction to Visual Culture
Introduction to Visual Culture
 
Pearl academy sample design aptitude test
Pearl academy sample design aptitude testPearl academy sample design aptitude test
Pearl academy sample design aptitude test
 
Irregular forms
Irregular  formsIrregular  forms
Irregular forms
 
Amrita shergil
Amrita  shergilAmrita  shergil
Amrita shergil
 
Chapter 9 pattern and texture
Chapter 9 pattern and textureChapter 9 pattern and texture
Chapter 9 pattern and texture
 
Nid solved dat question paper
Nid solved dat question paperNid solved dat question paper
Nid solved dat question paper
 
Elements of Design
Elements of DesignElements of Design
Elements of Design
 
Nift ug creative ability test
Nift ug creative ability testNift ug creative ability test
Nift ug creative ability test
 
Introduction to Visual Culture
Introduction to Visual CultureIntroduction to Visual Culture
Introduction to Visual Culture
 
Graphic Design 101
Graphic Design 101Graphic Design 101
Graphic Design 101
 
Incredible islamic art (geometry and mathematics)
Incredible islamic art (geometry and mathematics) Incredible islamic art (geometry and mathematics)
Incredible islamic art (geometry and mathematics)
 
Nid pgdpd product design sample paper
Nid pgdpd product design sample paperNid pgdpd product design sample paper
Nid pgdpd product design sample paper
 
Introduction to islamic architecture
Introduction to islamic architecture Introduction to islamic architecture
Introduction to islamic architecture
 
Geometry of Islamic Architecture
Geometry of Islamic ArchitectureGeometry of Islamic Architecture
Geometry of Islamic Architecture
 

Destacado

Masters in fashion management NIFT gat Sample paper
Masters in fashion management NIFT gat Sample paperMasters in fashion management NIFT gat Sample paper
Masters in fashion management NIFT gat Sample paperAcademy of Fashion & Design
 
NIFT Test series: Sample GAT paper of fashion design1
NIFT Test series: Sample GAT paper of fashion design1NIFT Test series: Sample GAT paper of fashion design1
NIFT Test series: Sample GAT paper of fashion design1Academy of Fashion & Design
 
nift question papers for fashion management - MAT1
nift question papers for fashion management - MAT1nift question papers for fashion management - MAT1
nift question papers for fashion management - MAT1Academy of Fashion & Design
 
How can Design Thinking be applied for Social Change?
How can Design Thinking be applied for Social Change?How can Design Thinking be applied for Social Change?
How can Design Thinking be applied for Social Change?Anel Palafox
 
Sustainable Design Part One: Building An Environmental Ethic
Sustainable Design Part One: Building An Environmental EthicSustainable Design Part One: Building An Environmental Ethic
Sustainable Design Part One: Building An Environmental EthicTerri Meyer Boake
 
Sustainable Design Part Two: Climate Related Issues
Sustainable Design Part Two: Climate Related IssuesSustainable Design Part Two: Climate Related Issues
Sustainable Design Part Two: Climate Related IssuesTerri Meyer Boake
 
17 Ways to Design a Presentation People Want to View
17 Ways to Design a Presentation People Want to View17 Ways to Design a Presentation People Want to View
17 Ways to Design a Presentation People Want to ViewJim MacLeod
 

Destacado (10)

Masters in fashion management NIFT gat Sample paper
Masters in fashion management NIFT gat Sample paperMasters in fashion management NIFT gat Sample paper
Masters in fashion management NIFT gat Sample paper
 
NIFT Test series: Sample GAT paper of fashion design1
NIFT Test series: Sample GAT paper of fashion design1NIFT Test series: Sample GAT paper of fashion design1
NIFT Test series: Sample GAT paper of fashion design1
 
nift question papers for fashion management - MAT1
nift question papers for fashion management - MAT1nift question papers for fashion management - MAT1
nift question papers for fashion management - MAT1
 
Nid pg solved question papers
Nid pg solved question papersNid pg solved question papers
Nid pg solved question papers
 
Sample paper-nift-fashion-management-gat1
Sample paper-nift-fashion-management-gat1Sample paper-nift-fashion-management-gat1
Sample paper-nift-fashion-management-gat1
 
Nift ug design gat question paper 2
Nift ug design gat question paper 2Nift ug design gat question paper 2
Nift ug design gat question paper 2
 
How can Design Thinking be applied for Social Change?
How can Design Thinking be applied for Social Change?How can Design Thinking be applied for Social Change?
How can Design Thinking be applied for Social Change?
 
Sustainable Design Part One: Building An Environmental Ethic
Sustainable Design Part One: Building An Environmental EthicSustainable Design Part One: Building An Environmental Ethic
Sustainable Design Part One: Building An Environmental Ethic
 
Sustainable Design Part Two: Climate Related Issues
Sustainable Design Part Two: Climate Related IssuesSustainable Design Part Two: Climate Related Issues
Sustainable Design Part Two: Climate Related Issues
 
17 Ways to Design a Presentation People Want to View
17 Ways to Design a Presentation People Want to View17 Ways to Design a Presentation People Want to View
17 Ways to Design a Presentation People Want to View
 

Similar a Nift old question paper masters in design gat

Similar a Nift old question paper masters in design gat (20)

Cat Qp
Cat QpCat Qp
Cat Qp
 
Tcs 2011with solutions
Tcs 2011with solutionsTcs 2011with solutions
Tcs 2011with solutions
 
class_vi_paper_1.pdf
class_vi_paper_1.pdfclass_vi_paper_1.pdf
class_vi_paper_1.pdf
 
Upcat math 2014
Upcat math 2014Upcat math 2014
Upcat math 2014
 
Riha
RihaRiha
Riha
 
Tcs questions papers
Tcs questions papersTcs questions papers
Tcs questions papers
 
MathsX.pdf
MathsX.pdfMathsX.pdf
MathsX.pdf
 
Cat paper (cat2)_www.fellowbuddy.com
Cat paper (cat2)_www.fellowbuddy.comCat paper (cat2)_www.fellowbuddy.com
Cat paper (cat2)_www.fellowbuddy.com
 
Qb maths
Qb mathsQb maths
Qb maths
 
General education-mathematics-3
General education-mathematics-3General education-mathematics-3
General education-mathematics-3
 
Arith1
Arith1Arith1
Arith1
 
Bank
BankBank
Bank
 
Qt sure shots
Qt sure shotsQt sure shots
Qt sure shots
 
Ppnet Numerical Aptitude Rrb Ii
Ppnet Numerical Aptitude Rrb IiPpnet Numerical Aptitude Rrb Ii
Ppnet Numerical Aptitude Rrb Ii
 
Capgemini quants and reasoning
Capgemini   quants and reasoningCapgemini   quants and reasoning
Capgemini quants and reasoning
 
Fourth level block 2 formal homework
Fourth level block 2 formal homeworkFourth level block 2 formal homework
Fourth level block 2 formal homework
 
Final exam review sheet # 3 2015
Final exam review sheet # 3 2015Final exam review sheet # 3 2015
Final exam review sheet # 3 2015
 
Hexaware mock test1
Hexaware mock test1Hexaware mock test1
Hexaware mock test1
 
TCS - sample questions
TCS - sample questionsTCS - sample questions
TCS - sample questions
 
Rrb tc patna
Rrb tc patnaRrb tc patna
Rrb tc patna
 

Más de Academy of Fashion & Design

Más de Academy of Fashion & Design (20)

Nid vijaywada 2015 prospectus
Nid vijaywada 2015 prospectusNid vijaywada 2015 prospectus
Nid vijaywada 2015 prospectus
 
Nid past test paper 2014 b.des
Nid past test paper 2014 b.desNid past test paper 2014 b.des
Nid past test paper 2014 b.des
 
Nid 2015 prospectus
Nid 2015 prospectusNid 2015 prospectus
Nid 2015 prospectus
 
Uceed 2015 question paper
Uceed 2015 question paperUceed 2015 question paper
Uceed 2015 question paper
 
Uceed 2015 brochure b.des iit mumbai
Uceed 2015 brochure b.des iit mumbaiUceed 2015 brochure b.des iit mumbai
Uceed 2015 brochure b.des iit mumbai
 
Uceed notification
Uceed notificationUceed notification
Uceed notification
 
Soft pune managerial ability test 4
Soft pune managerial ability test 4Soft pune managerial ability test 4
Soft pune managerial ability test 4
 
Soft pune managerial ability test 3
Soft pune managerial ability test 3Soft pune managerial ability test 3
Soft pune managerial ability test 3
 
Soft pune managerial ability test 21
Soft pune managerial ability test 21Soft pune managerial ability test 21
Soft pune managerial ability test 21
 
Soft pune general ability test 4
Soft pune general ability test 4Soft pune general ability test 4
Soft pune general ability test 4
 
Soft pune general ability test 3
Soft pune  general ability test 3Soft pune  general ability test 3
Soft pune general ability test 3
 
Soft pune general ability test 2
Soft pune general ability test 2Soft pune general ability test 2
Soft pune general ability test 2
 
Soft pune general ability test 1
Soft pune general ability test 1Soft pune general ability test 1
Soft pune general ability test 1
 
Soft pune creative ability test 4
Soft pune creative ability test 4Soft pune creative ability test 4
Soft pune creative ability test 4
 
Soft pune creative ability test 3
Soft pune creative ability test 3Soft pune creative ability test 3
Soft pune creative ability test 3
 
Soft pune creative ability test 2
Soft pune creative ability test 2Soft pune creative ability test 2
Soft pune creative ability test 2
 
Pearl academy ug general profficiency test 2014
Pearl academy ug general profficiency test 2014Pearl academy ug general profficiency test 2014
Pearl academy ug general profficiency test 2014
 
Pearl academy ug design aptitude test
Pearl academy ug design aptitude testPearl academy ug design aptitude test
Pearl academy ug design aptitude test
 
Pearl academy prospectus 2015
Pearl academy prospectus 2015Pearl academy prospectus 2015
Pearl academy prospectus 2015
 
Pearl academy pg sample question paper
Pearl academy pg sample question paperPearl academy pg sample question paper
Pearl academy pg sample question paper
 

Último

Merck Moving Beyond Passwords: FIDO Paris Seminar.pptx
Merck Moving Beyond Passwords: FIDO Paris Seminar.pptxMerck Moving Beyond Passwords: FIDO Paris Seminar.pptx
Merck Moving Beyond Passwords: FIDO Paris Seminar.pptxLoriGlavin3
 
Microsoft 365 Copilot: How to boost your productivity with AI – Part one: Ado...
Microsoft 365 Copilot: How to boost your productivity with AI – Part one: Ado...Microsoft 365 Copilot: How to boost your productivity with AI – Part one: Ado...
Microsoft 365 Copilot: How to boost your productivity with AI – Part one: Ado...Nikki Chapple
 
How to write a Business Continuity Plan
How to write a Business Continuity PlanHow to write a Business Continuity Plan
How to write a Business Continuity PlanDatabarracks
 
The State of Passkeys with FIDO Alliance.pptx
The State of Passkeys with FIDO Alliance.pptxThe State of Passkeys with FIDO Alliance.pptx
The State of Passkeys with FIDO Alliance.pptxLoriGlavin3
 
Decarbonising Buildings: Making a net-zero built environment a reality
Decarbonising Buildings: Making a net-zero built environment a realityDecarbonising Buildings: Making a net-zero built environment a reality
Decarbonising Buildings: Making a net-zero built environment a realityIES VE
 
[Webinar] SpiraTest - Setting New Standards in Quality Assurance
[Webinar] SpiraTest - Setting New Standards in Quality Assurance[Webinar] SpiraTest - Setting New Standards in Quality Assurance
[Webinar] SpiraTest - Setting New Standards in Quality AssuranceInflectra
 
Varsha Sewlal- Cyber Attacks on Critical Critical Infrastructure
Varsha Sewlal- Cyber Attacks on Critical Critical InfrastructureVarsha Sewlal- Cyber Attacks on Critical Critical Infrastructure
Varsha Sewlal- Cyber Attacks on Critical Critical Infrastructureitnewsafrica
 
Data governance with Unity Catalog Presentation
Data governance with Unity Catalog PresentationData governance with Unity Catalog Presentation
Data governance with Unity Catalog PresentationKnoldus Inc.
 
Glenn Lazarus- Why Your Observability Strategy Needs Security Observability
Glenn Lazarus- Why Your Observability Strategy Needs Security ObservabilityGlenn Lazarus- Why Your Observability Strategy Needs Security Observability
Glenn Lazarus- Why Your Observability Strategy Needs Security Observabilityitnewsafrica
 
Abdul Kader Baba- Managing Cybersecurity Risks and Compliance Requirements i...
Abdul Kader Baba- Managing Cybersecurity Risks  and Compliance Requirements i...Abdul Kader Baba- Managing Cybersecurity Risks  and Compliance Requirements i...
Abdul Kader Baba- Managing Cybersecurity Risks and Compliance Requirements i...itnewsafrica
 
A Framework for Development in the AI Age
A Framework for Development in the AI AgeA Framework for Development in the AI Age
A Framework for Development in the AI AgeCprime
 
Passkey Providers and Enabling Portability: FIDO Paris Seminar.pptx
Passkey Providers and Enabling Portability: FIDO Paris Seminar.pptxPasskey Providers and Enabling Portability: FIDO Paris Seminar.pptx
Passkey Providers and Enabling Portability: FIDO Paris Seminar.pptxLoriGlavin3
 
Arizona Broadband Policy Past, Present, and Future Presentation 3/25/24
Arizona Broadband Policy Past, Present, and Future Presentation 3/25/24Arizona Broadband Policy Past, Present, and Future Presentation 3/25/24
Arizona Broadband Policy Past, Present, and Future Presentation 3/25/24Mark Goldstein
 
Design pattern talk by Kaya Weers - 2024 (v2)
Design pattern talk by Kaya Weers - 2024 (v2)Design pattern talk by Kaya Weers - 2024 (v2)
Design pattern talk by Kaya Weers - 2024 (v2)Kaya Weers
 
New from BookNet Canada for 2024: Loan Stars - Tech Forum 2024
New from BookNet Canada for 2024: Loan Stars - Tech Forum 2024New from BookNet Canada for 2024: Loan Stars - Tech Forum 2024
New from BookNet Canada for 2024: Loan Stars - Tech Forum 2024BookNet Canada
 
Unleashing Real-time Insights with ClickHouse_ Navigating the Landscape in 20...
Unleashing Real-time Insights with ClickHouse_ Navigating the Landscape in 20...Unleashing Real-time Insights with ClickHouse_ Navigating the Landscape in 20...
Unleashing Real-time Insights with ClickHouse_ Navigating the Landscape in 20...Alkin Tezuysal
 
How to Effectively Monitor SD-WAN and SASE Environments with ThousandEyes
How to Effectively Monitor SD-WAN and SASE Environments with ThousandEyesHow to Effectively Monitor SD-WAN and SASE Environments with ThousandEyes
How to Effectively Monitor SD-WAN and SASE Environments with ThousandEyesThousandEyes
 
2024 April Patch Tuesday
2024 April Patch Tuesday2024 April Patch Tuesday
2024 April Patch TuesdayIvanti
 
Emixa Mendix Meetup 11 April 2024 about Mendix Native development
Emixa Mendix Meetup 11 April 2024 about Mendix Native developmentEmixa Mendix Meetup 11 April 2024 about Mendix Native development
Emixa Mendix Meetup 11 April 2024 about Mendix Native developmentPim van der Noll
 
How AI, OpenAI, and ChatGPT impact business and software.
How AI, OpenAI, and ChatGPT impact business and software.How AI, OpenAI, and ChatGPT impact business and software.
How AI, OpenAI, and ChatGPT impact business and software.Curtis Poe
 

Último (20)

Merck Moving Beyond Passwords: FIDO Paris Seminar.pptx
Merck Moving Beyond Passwords: FIDO Paris Seminar.pptxMerck Moving Beyond Passwords: FIDO Paris Seminar.pptx
Merck Moving Beyond Passwords: FIDO Paris Seminar.pptx
 
Microsoft 365 Copilot: How to boost your productivity with AI – Part one: Ado...
Microsoft 365 Copilot: How to boost your productivity with AI – Part one: Ado...Microsoft 365 Copilot: How to boost your productivity with AI – Part one: Ado...
Microsoft 365 Copilot: How to boost your productivity with AI – Part one: Ado...
 
How to write a Business Continuity Plan
How to write a Business Continuity PlanHow to write a Business Continuity Plan
How to write a Business Continuity Plan
 
The State of Passkeys with FIDO Alliance.pptx
The State of Passkeys with FIDO Alliance.pptxThe State of Passkeys with FIDO Alliance.pptx
The State of Passkeys with FIDO Alliance.pptx
 
Decarbonising Buildings: Making a net-zero built environment a reality
Decarbonising Buildings: Making a net-zero built environment a realityDecarbonising Buildings: Making a net-zero built environment a reality
Decarbonising Buildings: Making a net-zero built environment a reality
 
[Webinar] SpiraTest - Setting New Standards in Quality Assurance
[Webinar] SpiraTest - Setting New Standards in Quality Assurance[Webinar] SpiraTest - Setting New Standards in Quality Assurance
[Webinar] SpiraTest - Setting New Standards in Quality Assurance
 
Varsha Sewlal- Cyber Attacks on Critical Critical Infrastructure
Varsha Sewlal- Cyber Attacks on Critical Critical InfrastructureVarsha Sewlal- Cyber Attacks on Critical Critical Infrastructure
Varsha Sewlal- Cyber Attacks on Critical Critical Infrastructure
 
Data governance with Unity Catalog Presentation
Data governance with Unity Catalog PresentationData governance with Unity Catalog Presentation
Data governance with Unity Catalog Presentation
 
Glenn Lazarus- Why Your Observability Strategy Needs Security Observability
Glenn Lazarus- Why Your Observability Strategy Needs Security ObservabilityGlenn Lazarus- Why Your Observability Strategy Needs Security Observability
Glenn Lazarus- Why Your Observability Strategy Needs Security Observability
 
Abdul Kader Baba- Managing Cybersecurity Risks and Compliance Requirements i...
Abdul Kader Baba- Managing Cybersecurity Risks  and Compliance Requirements i...Abdul Kader Baba- Managing Cybersecurity Risks  and Compliance Requirements i...
Abdul Kader Baba- Managing Cybersecurity Risks and Compliance Requirements i...
 
A Framework for Development in the AI Age
A Framework for Development in the AI AgeA Framework for Development in the AI Age
A Framework for Development in the AI Age
 
Passkey Providers and Enabling Portability: FIDO Paris Seminar.pptx
Passkey Providers and Enabling Portability: FIDO Paris Seminar.pptxPasskey Providers and Enabling Portability: FIDO Paris Seminar.pptx
Passkey Providers and Enabling Portability: FIDO Paris Seminar.pptx
 
Arizona Broadband Policy Past, Present, and Future Presentation 3/25/24
Arizona Broadband Policy Past, Present, and Future Presentation 3/25/24Arizona Broadband Policy Past, Present, and Future Presentation 3/25/24
Arizona Broadband Policy Past, Present, and Future Presentation 3/25/24
 
Design pattern talk by Kaya Weers - 2024 (v2)
Design pattern talk by Kaya Weers - 2024 (v2)Design pattern talk by Kaya Weers - 2024 (v2)
Design pattern talk by Kaya Weers - 2024 (v2)
 
New from BookNet Canada for 2024: Loan Stars - Tech Forum 2024
New from BookNet Canada for 2024: Loan Stars - Tech Forum 2024New from BookNet Canada for 2024: Loan Stars - Tech Forum 2024
New from BookNet Canada for 2024: Loan Stars - Tech Forum 2024
 
Unleashing Real-time Insights with ClickHouse_ Navigating the Landscape in 20...
Unleashing Real-time Insights with ClickHouse_ Navigating the Landscape in 20...Unleashing Real-time Insights with ClickHouse_ Navigating the Landscape in 20...
Unleashing Real-time Insights with ClickHouse_ Navigating the Landscape in 20...
 
How to Effectively Monitor SD-WAN and SASE Environments with ThousandEyes
How to Effectively Monitor SD-WAN and SASE Environments with ThousandEyesHow to Effectively Monitor SD-WAN and SASE Environments with ThousandEyes
How to Effectively Monitor SD-WAN and SASE Environments with ThousandEyes
 
2024 April Patch Tuesday
2024 April Patch Tuesday2024 April Patch Tuesday
2024 April Patch Tuesday
 
Emixa Mendix Meetup 11 April 2024 about Mendix Native development
Emixa Mendix Meetup 11 April 2024 about Mendix Native developmentEmixa Mendix Meetup 11 April 2024 about Mendix Native development
Emixa Mendix Meetup 11 April 2024 about Mendix Native development
 
How AI, OpenAI, and ChatGPT impact business and software.
How AI, OpenAI, and ChatGPT impact business and software.How AI, OpenAI, and ChatGPT impact business and software.
How AI, OpenAI, and ChatGPT impact business and software.
 

Nift old question paper masters in design gat

  • 1. Academy of Fashion & Design Email: contact@afdindia.com Ph: 08800337062 Get into premier design school In India & Abroad with AFD  http://www.afdindia.com                       MASTERS IN DESIGN GENERAL ABILITY TEST Time allowed:- 2 hrs Total Question:- 150 This test comprises the following sub-tests. Quantitative ability English comprehension English Communication Analytical ability General Awareness Each question carries one mark. Answers are required to be marked only on the OMR/ICR Answer-Sheet, which shall be provided separately. For each question, four alternative answers have been provided out of which only one is correct. Darken the appropriate circle in the Answer-Sheet by using Ball Pen only on the best alternative amongst a), b), c) or d). Section-1-Quantitative ability Q1. Three bells commence tolling together and they toll after 0.25, 0.1 and 0.125 sec. After what interval will they again toll together? a) 1.5 sec b) 2.5 sec c) 0.5 sec d) 4.2 sec Q2. Find the smallest fraction which, when added to EMBED Equation.3 gives a
  • 2. whole number. a) 37 / 40 b) 35 / 39 c) 32 / 40 d) 45 / 35 Q3. If 5 = 2.24, then what is the value of EMBED Equation.3 ? a) 1.48 b) 1.58 c) 1.68 d) 1.89 Q4. Find the value of a 3 + b 3 + c 3 – 3abc when a + b + c = 9 and a 2 + b 2 + c 2 = 29. a) 9 b) 3 c) 27 d) 81 e) None of these Q5. If we write the numbers from 1 to 201, what is the sum of all the odd numbers? a) 10,201 b) 10,220 c) 9,102 d) None of these Q6. The sum of two numbers is 20, and their difference is 2 EMBED Equation.3 , find the ratio of the numbers. a) 8 : 10 b) 2 : 8 c) 9 : 7 d) 10 : 12 Q7. In an exam, the average was found to be 50 marks. After deducting computational errors the marks of the 100 candidates had to be changed from 90 to 60 each and the average came down to 45 marks. The total number of candidates who took the exam were: a) 300 b) 600 c) 200 d) 150 e) None of these Q8. A man buys milk at 60 P per liter adds one – third of water to it and sells the mixture at 72 P per liter. The profit per cent is _____________. a) 45% b) 60% c) 72% d) 75% Q9. Two equal amounts of money are deposited in two banks, each at 15 % per annum, for EMBED Equation.3 years and 5 years respectively. If the difference between their interests is Rs. 144 each sum is Rs __________. a) 460/- b) 500/- c) 640/- d) 720/- Q10. I give a certain person Rs. 8000 at simple interest for 3 years at EMBED Equation. 3 p.c. How much more should I have gained had I given it at compound interest? a) 148.45/- b) 138.38/- c) 134.89/- d) 134.25/-
  • 3. Q11. A sum of Rs. 6.40 is made up of 80 coins, which are either 10–paise or 5–paisa coins. How many are of 5 P.? a) 58 b) 64 c) 32 d) 25 Q12. A and B can finish a field work in 30 days, B and C in 40 days while C and A in 60 days. How long will they take to finish it together? a) 26 EMBED Equation.3 days b) 39 days c) 25 EMBED Equation.3 days d) 25 days Q13. If 3 men with 4 boys can earn Rs. 756 in 7 days, and 11 men with 13 boys an earn Rs. 3008 in 8 days, in what time will 7 men with 9 boys earn Rs. 2480? a) 8 days b) 5 days c) 12 days d) 10 days Q14. Pipe A can fill a tank in 20 hours while pipe B alone can fill it in 30 hours and pipe C can empty the full tank in 40 hours. If all the pipes are opened together, how much time will be needed to make the tank full? a) 7 EMBED Equation.3 hrs. b) 1 EMBED Equation.3 hrs. c) 17 EMBED Equation.3 hrs. d) 77 EMBED Equation.3 hrs. Q15. Compare the rates of two trains, one travelling at 45 km an hour and the other at 10 m a second. a) 5 : 4 b) 6 : 4 c) 2 : 6 d) 4 : 8 Q16. Find the length of a bridge which a train 130m long, travelling at 45 km an hour, can cross in 30 sec. a) 396 m b) 249 m c) 245 m d) 234 m Q17. A man can row 6 km/hr in still water. When the river is running at 1.2 km/hr, it takes him 1 hour to row to a place and back. How far is the place? a) 2.66 km b) 2.85 km c) 88.2 km d) 2.88 km Q18. If 2 liters of water are evaporated on boiling from 8 liters of sugar solution containing 5 % sugar, find the percentage of sugar in the remaining solution. a) 6 EMBED Equation.3 % b) 6 EMBED Equation.3 % c) 5 EMBED Equation.3 % d) 2 EMBED Equation.3 % Q19. The age of a man is 4 times that of his son. 5 yr. ago, the man was nine times as old as his son was at the time. What is the present age of the man? a) 64 yrs b) 49 yrs c) 32 yrs d) 45 yrs
  • 4. What is the greatest number that will divided 38,45 and 52 and leave as remainders 2,3 and 4 respectively? 5 b) 6 c) 7 d) 8 Find the greatest number, which will divide 410, 751 and 1030 so as to leave remainder 7 in each case? 30 b) 28 c) 31 d) 29 The numbers 11284 and 7655, when divided by a certain number of three digits, leave the same remainder. Find that number of three digits. 191 b) 130 c) 131 d ) 190 Find the greatest number of six digits which, on being divided by 6,7,8,9 and 10 leaves 4,5,6,7 and 8 as remainders respectively. 997918 b) 997920 c) 887920 d) 887918 Find the greatest number less than 900, which is divisible by 8,12 and 28.d 540 b) 640 c) 740 d ) 840 What greatest number can be subtracted form 10,000 so that the remainder may be divisible by 32, 36, 48 and 54? B 9186 b) 9136 c) 3156 d ) 8146 Find out whether (3x – 1) is a factor of 27x3 – 9x2 – 6x + 2 by the above rule. (3x-2) b) (3x-1) c) (2x-1) d ) (3x-2) What should be subtracted from 27x3 – 9x2 – 6x – 5 to make it exactly divisible by (3x – 1)? -5 b) -6 c) -7 d) -8 If x + _1_ = 2 , the value of x2 + _1_ = ? X X2 6 b) 5 c) 1 d) 2 If X + Y = 3, XY = 2; find the value of X3 – Y3 ? 6 b) 7 c) 3 d) 4
  • 5. If x = 12, y = 4; find the value of (x+y)x/y 4096 b) 3096 c) 2096 d ) 1096 Academy of Fashion & Design Email: contact@afdindia.com Ph: 08800337062 Get into premier design school In India & Abroad with AFD  http://www.afdindia.com                       Section-2- English Comprehensive Directions (Q31-35):- Read the passage and answer the questions that follow: Mrs. Loisel now learned what it was like to be really poor. She made up her mind to face it and played her part bravely. This terrible debt had to be paid and she would pay it. The maid was dismissed; the flat was given up and they moved into a garret. She did all the rough household work; washed up after meals and ruined her finger nails scrubbing dirty dishes and pans. She did all the washing and hung it out on the line to dry. Every morning, she carried the rubbish down to the street and brought the water, pausing for breath at the top of each flight of stairs. Dressed like a working woman, she went with her basket on her arm to the greengrocer, the grocer and the butcher, bargaining, arguing and fighting for every penny. Her husband spent his evenings, working at some shopkeepers’ account, and at night, he would often copy papers at a few pennies a page. Thus life went on for ten years. At the end of that time, they had paid off every thing to the last penny, including the interest on the loan.
  • 6. Mrs. Loisel now looked like an old woman. She had become a typical poor man’s wife, rough and coarse. Her hair was neglected, her dress was untidy, and her hands were red. But now and then, when her husband was at the office, she would sit by the window and her thoughts would go back to that far away evening, the evening of her beauty and her success. What would have been the ends of it if she had not lost the necklace? Who could say? How strange and varied are the chances of life. How small a thing can save or ruin you. One Sunday, she went for a walk in the Champa Elysees and she caught sight of a lady with a child. She recognized Mrs. Forestier, who looked as young, as pretty and as attractive as ever. Mrs. Loisel felt a wave of sadness pass over her. Should she speak to her? Why not? Now that the debt was paid, why should she not tell her the whole story? She went up to her “Good morning, Jeanne.” Her friend did not recognize her and said, “I’ m afraid I don’t know you; you must have made a mistake.” “No, I am Mathilde Loisel.” Her friend uttered a cry. ‘Oh’ my poor dear Mathilde! How you have changed!” “Yes, I have been through a very hard time since I saw you last.” “What do you mean?” “You remember the diamond necklace you lent me to wear?” “Yes. Well?” “Well, I lost it.” “I don’t understand. You brought it back to me.” “What I brought you back was another one, exactly like it and for the ten years, we have been busy paying for it. You must understand that it was not an easy matter for people like us, who hadn’t a penny? However, it’s all over now. I can’t tell you what a relief it is!” Mrs. Forestier stopped dead. “You mean that you bought a diamond necklace to replace mine?” “Yes. And you never noticed it? They were certainly very much alike,” She smiled with pride and satisfaction. But Mrs. Forestier seized both her hands in great distress. “Oh, my poor, dear Mathilde1 Why? Mine was only imitation. At the most it was worth five hundred francs!”
  • 7. Q31. Mrs. Loisel was not a well off lady but still, she took to pay back the huge amount of necklace a) In a half – hearted way b) In a reluctant manner c) Quite unwillingly d) In a determined way e) Under compulsion of law Q32. Entire money was paid up by her without excuse but a) It took ten years for Loisel to clear off the dues in installments b) It took ten years for them to save that much amount to pay back c) It took lesser period in paying back the money. d) They had not paid back in full the amount of necklace even during a period of ten years e) They were running short of a little amount to pay back the amount in full at the end of ten years. Q33. Mrs. Loisel struggled very hard to pay back the amount and it. a) Had made her weak and ill b) Had no effect on her health c) Had made her demoralised and ill humoured d) Had bereft her of her memory e) Had affected her looks Q34. When Mrs. Loisel met Mrs. Forestier she a) Told her that she had lost the necklace but promised to return another exactly like it b) Admitted that she had replaced Mrs. Forestier’s diamond necklace with a necklace of imitation diamonds. c) Told her that the necklace she had returned was not the same that she had borrowed d) Accused Mrs. Forestier of being a cause of ten years sufferings e) Begged her pardon to avoid talk about the necklace that has caused her so much worry Q35. Mrs. Forestier told Mrs. Loisel to the effect that. a) The necklace that was returned was made of imitation diamonds b) The necklace borrowed by Mrs. Loisel was not of real diamonds c) The necklace returned valued just 500 francs d) The necklace that she had returned valued less than 500 francs e) The necklace she had lent was of real diamonds Direction (Q36 - 40 ): Read the following passage carefully and answer the questions given below it.
  • 8. Imagine a ship built to sail amongst the ice at the North Pole. The ship has a hull two feet thick. This hull is shaped so that when the ice closes in on the ship she is pushed up – like a piece of soap in your hand and drifts in the top layer of the ice. A ship like this was built for the great Norwegian explorer Fridjof Nansen. One day, while he was walking by the sea, Nansen found a piece of Russian wood. Obviously the wood had drifted south across the Arctic Circle from Siberia. Nansen thought that if a piece of wood could make the journey so could he. If he had a ship which rested on the ice he would he able to drift across the Arctic Sea. And so that ‘Fram’ (which means forward) was built. In June 1893, Nansen left Oslo with a crew of 13 men and 30 dogs for the sledges. Many people thought his plans were mad and said that he was going to meet his death. By Sp. The ‘Fram’ had reached Cape Chelyuskin, the most northerly point of Siberia. Here, the ice became 30 ft thick and began to close in on the ‘Fram’. But just a Nansen had said the ship rose up, came to rest on the ice, and began to drift. For a year she drifted northwards, then slowly westwards. Nansen then decided to rush to the North Pole with his companion Hjelmar Johansen. At the time, no explorer had ever reached the Pole. They set off with 28 dogs, 3 sledges, 2 small boats, a tent, sleeping bags and food. By April 1895 they were within 200 miles of the Pole. This was nearer to the Pole than any other explorer had ever been. But conditions were terrible. The temperature dropped to 40 degrees below freezing point. The dogs became exhausted. Their cloths froze on their bodies. In the end they had to turn back. The two men and their dogs travelled for four months through ice and snow. Frozen, exhausted and with sinking spirits, they struggled on through mile after mile of snow. Once Nansen was nearly killed by animals. On another occasion, they tied their small boats to an iceberg and climbed up to make observations. The boats broke loose and began to drift. Nansen knew that they would not be able to go on without the boats. Still wearing this heavy cloths, he dived into the freezing water and swam after the boats. Then, after he had got them back, Nansen boat was nearly destroyed by an animal. At last, they reached Franz Josef Land, which is still inside the Arctic Circle. They decided to spend the winter there because it would have been dangerous to travel in a Pole winter. They built a hut of stone and covered it with animals’ skins and then, settled down for a long cold wait. They spent the winter hunting, talking and writing. In this way, they stopped themselves from going mad with loneliness.
  • 9. In May, when spring came, they set out again on their march Southwards. A month later, they walked into a party of British explorers. The British had been in the Arctic for two years and were waiting for their ship to take them home. The ship arrived and took Nansen and Johansen back to Norway. But what had happened to the “Fram”? There was no word. Her movements were as much a mystery as those of Nansen and Johansen had been. Astonishingly, a week after Nansen’s return, the ‘Fram’ drifted back to Norway – exactly three years after she had let. Nansen was given a hero’s welcome and so were the men of the ‘Fram’. Q36. For going on journey across the Arctic, Nansen got an idea of a particular ship, especially designed for the purpose because a) He must travel by a newly deigned ship to go on exploration mission. b) He must have a ship that would sail around the frozen Arctic. c) He must have a ship that would plough through the ice d) He must have a ship that would slide the top layer of the ice e) He must have a ship that would rest on the ice and drift. Q37. How did Nansen plan to go on exploration mission across the Arctic a) He planned to go to Russia across the Arctic Sea b) He planned to sail towards the North – Pole c) He planned to sail north – west toward the Pole d) He planned to proceed Northward on the ice e) He planned to go across the Arctic from north to south Q38. What happened when Nansen and Johansen tried to reach the North Pole proceeding westward? a) They lost the way and were misled b) They reached the Pole but could and stay there c) They reached the Pole and passed the winter there d) They approached closer to the Pole but were forced to keep away by weather extremities Q39. What happened to Nansen and Johansen while they were on return journey? a) They met unknown explorers who played host to them b) They met British explorers who offered them lift to Norway c) They met the British explorers who took them home in their ship d) They met the British explorers who took them home in their ship – the “Fram”
  • 10. e) They got a ship that had just landed a party of explorers and reached home. Q40. What happened to ‘Fram’ after it was abandoned by Nansen and Johansen? a) The ship was destroyed by the snowstorm b) Returned to Norway after a very long time c) Returned to Norway shortly after Nansen d) Nothing was heard of it e) The ship was remodeled by some other explorers and taken away by them. Directions: Below is given a passage followed by several possible followed by several possible inferences which can be drawn from the facts stated in the passage. You have to examine each inference separately in the context of passage and decide upon its degree of truth or falsity and mark: (a) if it is definitely true. (b) if it is probably true. (c) if the data are inadequate. (d) if it is probably false. (e) if it is definitely false. The serious accident in which a person was run over by a car yesterday has again focussed attention on the most unsatisfactory state of street lighting. No one expects side roads to be provided with the same standard of lighting, as a main road, but unless the council is prepared to make good its promise as regards road lighting, it will only be a question of time before there are further and perhaps fatal accidents. It seems that council has promised to improve the state of lighting on side roads. There will not be a single accident on roads if roads are satisfactorily illuminated. The accident that occurred was fatal. Several accidents have so far taken place because of unsatisfactory lighting. The accident occurred at night. Direction: Read the following passage and answer the question: There are several reasons for a headache. Physical, emotional and mental factors, anxiety and tension are a few. Sometimes, headache can be a signal of an underlying disease. More than medicines, yoga therapy eminently suits any need. Yoga is a comprehensive mode of culturing the body and the mind. Using an ‘Intergrated Approach of Yoga’, the yoga Research Centres have been able to cure some tough headaches. The integrated approach includes breathing, asanas, pranayama, meditation and devotional sessions. Yoga asanas, especially the ones imitating the natural postures of animals, have a tremendous tranquilising effect, without having t depend on common drugs. Pranayama inhibits random agitation in Pranic (energy) flows in Pranamayakosa, stabilising the
  • 11. autonomic nervous system. Dhyana and Samadhi culture the mind to relax it. This approach alters the reaction of an individual to headache. By interrupting the vicious cycle of pain – agony – pain, it prevents headache from becoming a crippling problem. Through asanas that calm you , the pranayama exercises that inhibit random energy flows and the meditation that cultivates and relaxes your mind, yoga offers a holistic from of pain relief. It stops you from becoming locked in the vicious circle of pain-anxiety- pain. Chronic pain: Chronic pain essentially is imbalance in prana (energy). This imbalance initially manifests only as functional abnormality like insomnia, lack of enthusiasm, fatigue, increased irritability and lack of concentration. Over the years, the imbalance settles in an organ. Chronic pain may sometimes be just a long standing muscle spasm, which later on amy give rise to organic changes in the form of chronic inflammation. The main theme of the passage is Headache Yoga Asanas Chronic Pain Relief from Headache Reasons for headache are Physical emotional and mental. Lack of concentration Too much of TV Carry any disease. What suits more than medicines? Asanas Homeopathy Integrated approach Yoga therapy What cultures the mind? Drugs Medicines Pain and agony Dhyana and samadhi What prevents headache from becoming a crippling problem? By attiring the approach By stabilizing the nervous system By interrupting the flow of energy. By interrupting the viscious cycle of pain – agony – pain
  • 12. Asanas calm you and Pranayama inhibit Viscous cycle of pain Stabilizes the mind Relaxes the mind Inhibits random energy flow. Yoga relaxes the mind and offer Holistic form of pain relief. Alters the reaction Dependence on common drugs To culture body and mind The disturbance in the energy level may lead to ____ Chronic pains Imbalance situation Pain and agony Distraction Chronic pain can sometimes be Fatigue Lack of concentration A long standing muscle spasm Organic changes / chronic inflammation For more than 3 decades, I achieved great success as a lawyer, till a stroke left my right side totally paralysed. Despite the doctor’s encouragement, I was consumed by rage and self- pity. I yearned to be active again. But what could a middle-age cripple like me do ? One day, glancing at some paintings I owned, I thought suddenly, “what about painting” ? In fact, I had always wanted to paint, but had never had the time. Now, I had plenty of time. In the last 25 years, I have completed 300 paintings—one of them appeared on the cover of the Reader’s Digest. The stroke, I realize, has helped me develop a latent talent and enjoy life. 55. Which one of the following is the correct statement ? While in the hospital, the author was— (A) angry with himself for falling ill. (B) relieved at the successful treatment. (C) frustrated at his helpless situation. (D) resentful at being hospitalized. 56. Why did the author consider himself a cripple? (A) He could not go back to work. (B) He could no longer use his right hand.
  • 13. (C) He could not use his time properly. (D) He could not lead an active life. 57. Which one of the following is the correct statement ? The paralytic stroke helped the author— (A) to face challenges in life successfully. (B) to realize his latent talent. (C) to learn a new hobby. (D) to earn more money. PASSAGE “Westward Ho !” we shouted as the sail of our crudely constructed raft, the Kon-Tiki caught the wind. The sail quickly filled and the Kon-Tiki began to move. The six of us were off to our great adventure. As night fell, the troughs of the sea grew gradually deeper and our first duel with the elements began. Each time we heard the sudden deafening hiss of a roller closeby and saw a white crest come towards us out of the darkness, we held on tight and waited for the worst. But invariably the Kon-Tiki calmly swung up her stern and rose skyward unperturbed. 58. What does the word ‘duel’ in the passage mean ? (A) A battle (B) A fortification (C) A two-side contest (D) Divided in two 59. Which one of the following is correct ? When big waves struck the raft the six people in it. (A) Started crying. (B) Showed courage and patience. (C) Acted in a rash manner. (D) Showed passiveness. 60. How was the Kon-Tiki’s performance on the high seas ? (A) Very shaky (B) Extremely poor (C) Stable and resolute (D) Unpredictable
  • 14. Academy of Fashion & Design Email: contact@afdindia.com Ph: 08800337062 Get into premier design school In India & Abroad with AFD  http://www.afdindia.com                       Section-3 English Communication Directions (Q.61-65): Choose the alternative, which is same in meaning to the key word:- AURICULAR a) Nose b) Eyes c) Ears d) Heart AVOWAL a) Sacred oath b) Open declaration c) Harsh Sound d) Stern denial COGNATE a) Known b) Of common origin c) Wheel shaped d) Substantial FRET a) Worry b) Behave c) Pine-heartedd) Regret JOWL a) Jaw b) Fiesta c) Bellowing d) Small Jar Directions (Q.66 - 70): Choose the correct one, which is opposite in meaning to the key word:- WARILY a) Negligently b) Timidly c) Vigilantly d) Noisily INTIMATE a) Proclaim b) Personal c) Apparel d) Preclude MODISH a) Vulgar b) Unfashionable c) Shabby d) Immodest
  • 15. WAG a) Critic b) Skunk c) Wit d) Dullard PROLIX a) Clever b) Punctual c) Charitable d) Precise Directions (Q.71 - 75): Choose the appropriate preposition: Mike has been ill ________ a long time. He has been in hospital ________ Oct. a) on b) to c) since d) for The office is very small. There’s space only ______ a desk and chair. a) to b) since c) for d) on Sue went ________ Mexico last year. a) for b) since c) to d) on Don’t open the door of the train __________ the train stops.. a) for b) Until c) Since d) on Where have you been? I’ve been waiting for you _________ twenty minutes. a) for b) Until c) Since d) to Directions (Q76-80):- Some parts of each of the following sentences are underlined. Either one underlines part is not acceptable in standard English or there is not error. With regard to error you are concerned with the underlined part only. EMBED Equation.3 Ram, Sham, Hari EMBED Equation.3 differences EMBED Equation.3 EMBED Equation.3 settled. EMBED Equation.3 . EMBED Equation.3 ask EMBED Equation.3 principal if I EMBED Equation.3 have one day EMBED Equation.3 . EMBED Equation.3 . EMBED Equation.3 machine worked EMBED Equation.3 EMBED Equation.3 many days and we EMBED Equation.3 rest at all. EMBED Equation.3 . Trios EMBED Equation.3 EMBED Equation.3 boys EMBED Equation.3 singing EMBED Equation.3 well that everyone was happy. EMBED Equation.3 .
  • 16. EMBED Equation.3 principal EMBED Equation.3 with his colleagues EMBED Equation.3 in EMBED Equation.3 car. EMBED Equation.3 . FILL IN THE BLANKS:- (81-90) My father is not a t home _________ a) presently b) first now c) Nil I have read ___________ book of this library a) each b) every c) Nil He is stronger than anybody _________ a) else b) here c) Nil __________ European likes to trand much a) an b) a c) Nil My young son goes to _______ school a) an b) a c) Nil I have got ___________ good health a) an b) the c) Nil I passed my holidays _________ the village of my uncle a) at b) in c) Nil He will come back ________ a week a) Nil b) within c) in Divide this apple ___________ the two boys. a) between b) among c) Nil The hunter shot the bird _________ a gun, a) with b) by c) from
  • 17. Academy of Fashion & Design Email: contact@afdindia.com Ph: 08800337062 Get into premier design school In India & Abroad with AFD  http://www.afdindia.com                       Section 4- Analytical Ability Directions (Q. 91 –95): Read the information and answer the questions given below the information There is a group of six persons – A, B, C, D, E & F in a family. They are psychologist Manager, Lawyer, Jeweler, Doctor and Engineer. The doctor is the grandfather of F. Who is a Psychologist. The Manager D is married to A. C is the Jeweler is married to a Lawyer. B is the mother of F and E. There are two married couples in the family. What is the profession of A? a) Doctor b) Engineer c) Psychologist d) Manager e ) None of these How is A related to E? a) Father b) Grand Father c) Mother d) Sister e ) None of these How many male members are there in the family? a) 3 b) 4 c) 2 d) information insufficient e) None of these What is the profession of E? a) Doctor b) Engineer c) Psychologist d) Manager e ) None of these Which of the following is one of the married couple?
  • 18. a) A-B b) A-C c) A-D d) F-E e ) None of these Direction (Q96-100): These question consist of a question and two statements numbered I and II given below it. You have to decide whether the data provided in the statements are sufficient to answer the question. Read both the statements and give your answer as follows: If the data in statement I alone is sufficient to answer the question If the data in statement II alone is sufficient to answer the question If the data in either statement I or II is sufficient to answer the question. If the data in statement I and II together are not sufficient to answer the question If the data in statement I and II together are necessary to answer the question Who scored the highest among A,B,C,D and E? B scored more than D, but not as much as C. E scored more than C but not more than A. a) B b) A c) D d) C e) E How many boys are there in the class? Mita’s rank among the girls is 5th from the top and her rank in the class is 9th from the bottom. Number of boys in the class is twice the number of girls. a) C b) A c) B d) D e) E Who is the immediate right of P among five persons P, Q, R, S and T facing North? R is third ot the left of Q and P is second to the right of R. Q is the immediate left of T who is second to the right of P. a) C b) A c) B d) D e) E Z is in which direction with respect to X? Y is to the South of x and Z is to the East of P which is to the North of Y P is to the South of x a) D b) C c) B d) A e) E How is P related to N? N is sister of M, who is son of Q whose wife is P. M is brother of N and son of Q whose wife is P. a) C b) D c) B d) A e) E
  • 19. Pointing to a photograph, a person tells his friend “She is the grand daughter of the elder brother of my father ”. How is the girl related to this man in the photograph a) Niece b) Daughter c) Daughter-in- law d) Mother e) None of these Direction (Q102 - 105): These questions are based on the following diagram in which the triangle represents the uneducated person, Circle represents young person, and rectangle represent employed person. Which number represents young uneducated & employed person ? a) 4 b) 3 c) 5 d) 2 e) None of these Which number represents educated, employed and young person. a) 2 b) 3 c) 1 d) 4 e ) None of these Which number represents young, educated and unemployed person? a) 3 b) 1 c) 7 d) 6 e ) None of these Which number represents young uneducated & unemployed person? (Check young should be there or not) a) 3 b) 5 c) 6 d) 4 e ) None of these Direction (Q106 - 110):- Answer these questions referring to the symbol-letter number sequence given below: NOPQYBZARSHIJKLMTUVGFEWXDC If letters of the above given series are written in reverse order then which letter will be third to the left of eighteenth letter from your right? a) Z b) F c) I d) L e ) None of these
  • 20. What will come in place of question mark (?) in the following series? NDP, QWB ,ZFR ? a) SVJ b) AFS c) IVS d) SFA e ) None of these Which of the following is fifth to the right of thirteen letter from your left? a) U b) J c) S d) Z e ) None of these If every alternate letter starting from O is replaced with odd numbers starting from 1, which letter or number will be third to the left of tenth letter from your right? a) 15 b) L c) K d) 13 e ) None of these If it is possible to make a meaningful word from the eighth, sixteenth, seventeenth and twenty-second letters from your left in the given series, which will be the first letter of that word? If no such word an be formed your answer would be X, and if more than one such word can be formed answer is P? a) M b) T c) X d) P e ) None of these In row of 16 boys when Karan was shifted by 2 places towards the left he became 7th from the left end. What was his earlier position from the right ? a) 8th b) 9th c) 7th d) 10th e) None of these Direction (Q 112-113):- In each of the following find the figure from the Answer set (i.e. 1, 2, 3, 4 and 5 ) which will continue the series given in the problem set (i.e. A, B, C, D, and E) PROBLEM FIGURES ANSWER FIGURES A B C D E 1 2 3 4 5 a) 2 b) 3 c) 1 d) 5 e ) None of these
  • 21. PROBLEM FIGURES ANSWER FIGURES A B C D E 1 2 3 4 5 a) 5 b) 3 c) 4 d) 1 e ) None of these If south-east becomes north, north-east becomes west and so on, what will west become? a) north-east b) north-west c) south-east d) south-west e) None of these A bus for Delhi leaves every forty minutes from a bus stand. An inquiry clerk told a passenger that the bus had already left ten minutes ago and the next bus would leave at 9.45 a.m. At what time did the inquiry clerk give this information to the passenger? a) 9.05 am b) 9.15 am c) 9.25 am d) 9.35 e) None of these Directions (Q116 - 120): Each of the following questions consists of five figures marked A,B, C, D and E called the Problem Figures followed by five other figures marked 1, 2, 3, 4, and 5 called the Answer Figures. Select a figure from amongst the Answer Figures which will continue the same series as established by the five Problem Figures.
  • 22. Academy of Fashion & Design Email: contact@afdindia.com Ph: 08800337062 Get into premier design school In India & Abroad with AFD  http://www.afdindia.com                       Section 5- General Awareness Which India State advertise itself as “ Never out of season”? kerela Himachal Uttaranchal Goa Which airline has a heart painted at the entry of each of its aircraft cot the company HQ is at lone field in Dallas? Quantas Gulf Airways Southwest Airlines Air china. Which memorable character made his appearance on the comic strip on October 4th 1950. Mickey Mouse Snoopy Fan tom Batman
  • 23. The ‘Jeans that Built America’? Diesel Levi’s Mudd Lee Find the odd one out Kathakali Mohini Attam Bharat Natyam Chakri The first cotton mill of India was set up at ……. Surat Mumbai Kanpur Ahmedabad Which of the following is the capital of “the land of midnight sun”? Vienna Oslo Helsinki Pragne Where would you find the Indian Textile Institute? Surat KAnpur Pune Mysore The study of the heavenly bodies is a) Astronaut’s b) Astronomy c) Astrology d) Aewnautics 130. World Literacy Day is on? a) Feb. 8th b) Jan. 18th c) Sept. 8th d) Feb. 18th
  • 24. 131. World Trade Organisation (WTO) came into existence in: a) 1996 b) 1998 c) 1995 d) 1997 132. The World Cup Cricket is held once in every a) 4 years b) 5 years c) 2 years d) 3 years 133. Baseball is the national game of a) England b) Kenya c) USA d) South Africa 134. The Flag of which colour is used by an enemy while surrendering? a) White b) Black c) Red d) Yellow 135. Which of the following ranks the highest in the Indian Army? a) Colonel b) Lieutenant General c) Major d) Captain 136. According to Hindu Mythology, who is the creator of the word? a) Brahma b) Vishwakamla c) Vishnu d) Mahesha 137. Which one of the following is not an ancient scripture? a) Rigveda b) Mundaka c) Upnishada d) Arthashastra 138. Camel can store extra water in the_______. a) Rumen b) Coelom c) Stomach d) None of these 139. The smallest State in area is: a) Kerala b) Tripura c) Punjab d) Goa 140. ‘Eat Healthy Think Better’? Tagline of which company. Parle Brittania Priya Gold Cherry merry 141. Tradus.in has launched an online book store at TradusBooks. in, offering book enthusiasts a wide assortment of titles, ease of shopping and free shipping of books
  • 25. across India. Tradus.in is a sister concern company of … a. Ibibo Web Pvt Ltd b. Yahoo c. Times Shopping d. 160by2.com 142. Harry Potter ends up marrying a.Hermoine b.Ron c.Gennie d.Snape 143. What is the amount that Future Group’s unit Pantaloon Retail planning to raise from its IPO? a. ` 600 cr b. ` 650 cr c. ` 700 cr d. ` 750 cr 144. HDFC Trustee Co Ltd acquired 2.44 per cent stake in which textiles and apparels maker company in August? a. Raymond b. Grasim c. Peter England d. Armani 145. How many circles have been allotted to Vodafone for 3G Mobile services? a. 9 b. 11 c. 14 d. 17 146. Spice Jet has been acquired by… a. Reliance ADA Group b. Mahindra n MahindraGroup c. Sun Group
  • 26. d. Larsen & Tourbo 147. Recently, which one of the following was included in the UNESCO's World Heritage list? a. Dilwara Temple b. Kalka-Simla Railway c. Bhiterkania Mangrove Area d. Vishakapatnam to Araku valley railway line 148. Which of the following is also known as “ Silicon valley of India”? a. Hyderabad b. Bangalore c. Chennai d. Noida 149. Peace Palace is the official headquarters of.......... a. WHO b. FAO c. ICJ d. IBRD 150. Find the odd one out. a. DOS b. WINDOWS c. LINUX d. LISP